298 research outputs found

    On a conjecture of Pomerance

    Full text link
    We say that k is a P-integer if the first phi(k) primes coprime to k form a reduced residue system modulo k. In 1980 Pomerance proved the finiteness of the set of P-integers and conjectured that 30 is the largest P-integer. We prove the conjecture assuming the Riemann Hypothesis. We further prove that there is no P-integer between 30 and 10^11 and none above 10^3500.Comment: 10 pages. Submitted to Acta Arithmetic

    Становлення і розвиток православних духовних семінарій на Правобережній Україні (кінець ХVІІІ – перша половина ХІХ ст.)

    Get PDF
    In this note we prove results of the following types. Let be given distinct complex numbers satisfying the conditions for and for every there exists an such that . Then If, moreover, none of the ratios with is a root of unity, then The constant −1 in the former result is the best possible. The above results are special cases of upper bounds for obtained in this paper

    On the rationality of Cantor and Ahmes series

    Get PDF
    AbstractWe give criteria for the rationality of Cantor series and series where a1, a2, ··· and b1, b2, ··· are integers such that an > 0 and the series converge. We precisely say when is rational (i) if {an}n=1∞ is a monotonic sequence of integers and bn + 1 − bn = o(an + 1) or lim and (ii) if for all large n. We give similar criteria for the rationality of Ahmes series and more generally series . For example, if bn > 0 and lim , where An = lcm(a1, a2, ···, an), then is rational if and only if for large n.On the other hand, we show that such results are impossible without growth restrictions. For example, we show that for any integers d > c > 1 there is a sequence {bn}n = 1∞ such that every number x from some interval can be represented as with an ∈ {c,d} for all n

    Multivariate Diophantine equations with many solutions

    Full text link
    We show that for each n-tuple of positive rational integers (a_1,..,a_n) there are sets of primes S of arbitrarily large cardinality s such that the solutions of the equation a_1x_1+...+a_nx_n=1 with the x_i all S-units are not contained in fewer than exp((4+o(1))s^{1/2}(log s)^{-1/2}) proper linear subspaces of C^n. This generalizes a result of Erdos, Stewart and Tijdeman for m=2 [Compositio 36 (1988), 37-56]. Furthermore we prove that for any algebraic number field K of degree n, any integer m with 1<=m<n, and any sufficiently large s there are integers b_0,...,b_m in a number field which are linearly independent over the rationals, and prime numbers p_1,...,p_s, such that the norm polynomial equation |N_{K/Q}(b_0+b_1x_1+...+b_mx_m)|=p_1^{z_1}...p_s^{z_s} has at least exp{(1+o(1)){n/m}s^{m/n}(log s)^{-1+m/n}) solutions in integers x_1,..,x_m,z_1,..,z_s. This generalizes a result of Moree and Stewart [Indag. Math. 1 (1990), 465-472]. Our main tool, also established in this paper, is an effective lower bound for the number of ideals in a number field K of norm <=X composed of prime ideals which lie outside a given finite set of prime ideals T and which have norm <=Y. This generalizes a result of Canfield, Erdos and Pomerance [J. Number Th. 17 (1983), 1-28], and of Moree and Stewart (see above).Comment: 29 page

    On conjectures and problems of Ruzsa concerning difference graphs of S-units

    Full text link
    Given a finite nonempty set of primes S, we build a graph G\mathcal{G} with vertex set Q\mathbb{Q} by connecting x and y if the prime divisors of both the numerator and denominator of x-y are from S. In this paper we resolve two conjectures posed by Ruzsa concerning the possible sizes of induced nondegenerate cycles of G\mathcal{G}, and also a problem of Ruzsa concerning the existence of subgraphs of G\mathcal{G} which are not induced subgraphs.Comment: 15 page

    Mooie en afwisselende carrière

    Get PDF

    Convergence properties of linear recurrence sequences

    Get PDF
    Analysis and Stochastic
    corecore